LSAT and Law School Admissions Forum

Get expert LSAT preparation and law school admissions advice from PowerScore Test Preparation.

 lsatscholar
  • Posts: 1
  • Joined: Sep 18, 2016
|
#28735
The conclusion here is that: Athletes who need to improve muscle strength should not consume engineered food.

Premises: Hormones produce growth in connective tissues rather than in muscle mass, which does not improve muscular strength

So, I was stuck between answer choices "A" and "C"

I tried the negation test and thought without both of them the argument falls apart.

I chose "A" because "C" was too strong and thought it was an sufficient assumption.

I'm having a hard time eliminating "A" because, I'm assuming if muscle mass does not increase strength, the premises of the argument falls apart and thus breaks the conclusion.
 Adam Tyson
PowerScore Staff
  • PowerScore Staff
  • Posts: 5153
  • Joined: Apr 14, 2011
|
#28763
A almost had me for a while, too, but in the end you and I should eliminate it for two reasons: 1) because it is based on a mistaken reversal of an implied conditional claim in the stimulus, and 2) because it relies on a causal relationship where none was posited.

The author tells us that the product doesn't produce growth in muscle mass, and that this means it doesn't improve strength. Think of that conditionally - if no growth in mass, no improvement in strength. The contrapositive must also be true - if it improves strength, it increases muscle mass - but answer A actually scrambles that up and gives us the reversal, that muscle growth is sufficient for strength.

If you prefer, try looking at it causally - the argument says that in the absence of a certain condition (muscle growth), we get the absence of another (strength). From that correlation he presumes that the presence of the first condition will cause the presence of the second - causation implied from correlation, which is never a good argument on the LSAT.

Finally, let's take another look at the negation technique as applied to answer A here. What if an increase in muscle mass does not improve strength? What impact does that have on the conclusion that athletes should avoid these products? None at all - the author has told us that the products do not improve strength, and this negation doesn't do anything to contradict that claim. There's nothing about the negation to suggest that maybe an athlete could get a benefit from using the product. The negation of the correct answer should pretty completely wreck the argument, and this does little to nothing to accomplish that goal.

Negate C, however, and you get what you need - even if the product doesn't increase strength, it still has some substantial benefits for athletes. That pretty much destroys the argument that athletes should not consume them!

I hope that helps some!
 PB410
  • Posts: 39
  • Joined: Apr 01, 2017
|
#39287
I'm not quite sure how to understand how answer choice c can be a right answer. The conclusion is about improving muscle strength, and how athletes should not consume engineered foods if they need to improve muscle strength. Answer choice C suggests there are other reasons why an athlete may benefit from consuming engineered food; but the conclusion is specifically about increasing muscle strength. The conclusion is not speaking to any other benefits outside of improving muscular strength. Any thoughts?
 Adam Tyson
PowerScore Staff
  • PowerScore Staff
  • Posts: 5153
  • Joined: Apr 14, 2011
|
#39579
Actually, PB410, the conclusion is just that athletes should not consume engineered foods. It's not about muscle strength, other than by implying that there is no other reason to consider consuming such foods. Answer C says "but there IS another reason!", and that is enough to pretty much destroy the claim that athletes should not consume them. Maybe they also need connective tissue to grow? Maybe they could benefit from the vitamins and minerals in the products? Maybe the amino acids also prevent cancer, or improve brain function, or make your teeth white and your hair shiny? Any of these would wreck the claim that athletes should not consume these foods, and so C must be an assumption of the argument.

Athletes obviously shouldn't bother consuming these foods if the ONLY reason they would do so is to improve muscle strength, but that is not what the argument is saying. Rather, that appears to be what the argument is assuming.

I hope that makes it a little more clear for you! Good luck with your continued studies!
 akanshalsat
  • Posts: 104
  • Joined: Dec 20, 2017
|
#47716
Hello! I see why and how C can be correct, but I just dont understand how a lsat taker, under the constraints of time, would be able to clearly eliminate A as being a strong contender. The conclusion straight up says that those athletes who need to improve their muscular strength should NOT consume engineered foods, with the evidence that it does NOT produce growth in muscle mass, only connective tissue. This assumes the muscle mass then is important in producing an increase in the strength of those athletes who want to do just that. I think C is a good answer, but since the conclusion was SPECIFIC to muscular strength, I felt A made more sense between the two.
 James Finch
PowerScore Staff
  • PowerScore Staff
  • Posts: 943
  • Joined: Sep 06, 2017
|
#49080
Hi Akansha,

This is a Supporter Assumption question, so we're looking for something new in the conclusion that we must then logically link to the existing premises. These questions, just like Justify questions, tend to yield great Prephrases.

The issue with answer choice (A) is that it's dealing with a red herring: muscle mass isn't the issue, it's whether we can conclude that something shouldn't be consumed because it doesn't increase muscle strength. We need to create a logical linkage between a food not increasing muscle strength and thus not consuming that food, or:

Increase Muscle Strength (IMS) :arrow: Consume Food (CF)

(C) gives us exactly that, while (A) would look like:

Increased Muscle Mass (IMM) :arrow: IMS

Which isn't anything like what we need. When in doubt, use the Assumption Negation technique to test assumption answer choices, and that will yield the correct one.

Hope this helps!
User avatar
 sdb606
  • Posts: 78
  • Joined: Feb 22, 2021
|
#87818
I eliminated C because I thought it was drawing too strong a conclusion. I would have picked it if it had said, "If an engineered food does not improve muscle strength, there is no other substantial advantage to athletes who need to increase their muscle strength from consuming it." If I negate C, the author could say, "That's true that there are other reasons for taking engineered foods but they don't apply to athletes who need to improve muscle strength."

!C does not destroy the argument.
 Robert Carroll
PowerScore Staff
  • PowerScore Staff
  • Posts: 1787
  • Joined: Dec 06, 2013
|
#87841
Sdb,

The conclusion in the last sentence is not limiting itself to athletes who need to improve their muscular strength. It's applying to all athletes. The phrase "who need to improve their muscular strength" is set off by commas because it's an appositive; it's not restricting the kind of athlete it's talking about, but making a comment it thinks is true of all athletes (and of course that comment explains why the author thinks athletes in general should not consume engineered foods).

The author would agree that all athletes need to improve their muscular strength. So there are no athletes who don't have to do that, so the restriction you want from the answer isn't necessary.

Robert Carroll

Get the most out of your LSAT Prep Plus subscription.

Analyze and track your performance with our Testing and Analytics Package.